2001 AIME II Problems/Problem 3

Revision as of 11:25, 28 February 2008 by 1=2 (talk | contribs)

Problem

Given that

$\begin{eqnarray*}x_{1}&=&211,\\ x_{2}&=&375,\\ x_{3}&=&420,\\ x_{4}&=&523, \textrm{ and}\\ x_{n}&=&x_{n-1}-x_{n-2}+x_{n-3}-x_{n-4}\textrm{ when }n\geq5, \end{eqnarray*}$ (Error compiling LaTeX. Unknown error_msg)

find the value of $x_{531}+x_{753}+x_{975}$.

Solution

$x_5=x_4-x_3+x_2-x_1$

$x_6=x_4-x_3+x_2-x_1-x_4+x_3-x_2=-x_1$

$x_7=-x_1-x_4+x_3-x_2+x_1+x_4-x_3=-x_2$

$x_8=-x_2+x_1+x_4-x_3+x_2-x_1-x_4=-x_3$

$x_9=-x_3+x_2-x_1-x_4+x_3-x_2+x_1=-x_4$

And it cycles back to $x_{11}=x_1$

Therefore, $x_{y}=x_{y-10}$, so

\[x_{531}+x_{753}+x_{975}=x_1+x_3+x_5=x_1+x_3+x_4-x_3+x_2-x_1=x_4+x_2=523+420=\boxed{943}\]

See also

2001 AIME II (ProblemsAnswer KeyResources)
Preceded by
Problem 2
Followed by
Problem 4
1 2 3 4 5 6 7 8 9 10 11 12 13 14 15
All AIME Problems and Solutions